CARDIOVASCULAR ACTUAL EXAM PAPER
2026 COMPLETE QUESTIONS AND
ANSWERS GRADED A+
⩥ A 45-year-old man presents with chest pain radiating to the left arm,
diaphoresis, and shortness of breath. His ECG shows ST-segment
elevation. What is the initial management response? Answer: Administer
aspirin and call for emergency medical services
⩥ An adult patient must maintain an International Normalized Ratio
(INR) between 2.0 and 3.0. The patient goes to a clinic for INR
determination, and the result is 1.4. Which of the following would be
likely to decrease the effects of warfarin (Coumadin)? Answer: Broccoli
Rationale: Warfarin (Coumadin) therapy that is a vitamin K antagonist
and can be counteracted by a patient who is eating a dietary intake of
vitamin K. Leafy green vegetables such as broccoli and spinach are
notorious for increasing vitamin K levels, and therefore inhibiting the
anticoagulation effect of warfarin. It's also important to remember that
with the direct oral anticoagulants category, vitamin K is not affected,
and therefore no dietary restrictions are required for this class of
medications such as apixaban, rivaroxaban, and edoxaban, which is a
distinct benefit over warfarin as noted above. Also, with warfarin it is
important to remember that the effects are based on the free drug, not the
protein bound drug level, so patients who have considerable protein
, stores will require more warfarin than those who are emaciated or have
low protein levels chronically.
With respect to this particular question, red wine and grapefruit do not
have an effect of lowering the INR, in fact, they will raise it by their
unique mechanisms. Grapefruit specifically causes warfarin levels to rise
through the CYP 450 system, and red wine causes the INR to increase
by thinning the blood and and red meat is not likely to have a
considerable impact, although it might have some impact if the protein
stores are otherwise low prior to initiating the red meat in the diet.
⩥ As a f/u from a hospitalization an adult patent presents with ankle
edema. Which of the following medications is the most likely cause of
the edema? Answer: Norvasc
Rationale: The most common side effects of calcium channel blockers
include constipation and lower extremity edema. The other options do
not have any relationship specifically with edema, in fact,
hydrochlorothiazide specifically reduces edema via diuresis.
⩥ A nurse practitioner places a 76-year-old patient on nifedipine
(Procardia) 10 mg t.i.d. for angina. The patient is unable to remember to
take the medication at the scheduled times. The practitioner should:
Answer: Change the dose to extended release 30 mg daily